¿Es realmente necesario un monopolo magnético para la cuantificación de carga?

El comentario habitual que ocurre en un primer encuentro de monopolos de Dirac es que resuelve el problema de la cuantificación de la carga eléctrica. También he estudiado los monopolos de t'Hooft Polyakov que cuantifican asintóticamente la carga. Básicamente prueban que mi gramo k Z dónde k es un número adimensional, mi es la carga eléctrica y gramo es la carga magnética.

En primer lugar, ¿a qué nos referimos cuando decimos que la carga debe cuantificarse? ¿Estamos tratando de decir que todas las cargas eléctricas que encontramos en la naturaleza son múltiplos enteros de la carga electrónica ( mi = 1.6 × 10 19 C )? En ese caso, ¿no es la explicación obvia el hecho de que todo está formado por electrones y protones? ¿Por qué tomarse la molestia de inventar monopolos para explicar este hecho?

Tal vez estemos tratando de explicar por qué la carga electrónica es ese número en particular. Seguramente, el tu ( 1 ) La carga puede ser cualquier número real. Pero no veo cómo la cuantización de la carga eléctrica ayuda a explicar este número de todos modos. En cambio, plantea más preguntas. Si tu ( 1 ) carga está cuantizada, es decir, que se permite que existan varios múltiplos enteros de un cuanto de carga, ¿dónde están todas las demás partículas elementales con todas las cargas múltiples integrales que se permiten? ¿Cuántas veces el cuanto de carga es la carga electrónica? ¿Qué partícula tiene la carga mínima permitida? Si resulta que esa partícula es el electrón (¿y por qué?), y usted va a explicar las cargas de otros compuestos de electrones en términos de la carga electrónica, ¿por qué molestarse en inventar la maquinaria del monopolo magnético en primer lugar?

+1. Creo que el punto de vista interesante sobre los monopolos magnéticos no se trata tanto de la cuantificación de carga "directa", sino que si tiene dos cargas cuya relación es irracional, entonces el grupo de calibre para el electromagnetismo sería R y no U (1).
Relacionado: physics.stackexchange.com/q/97909/2451 y enlaces allí.

Respuestas (1)

  1. Sí, "cuantificación de carga" significa que todas las cargas son un múltiplo de alguna unidad de carga fundamental mi .

  2. Por supuesto, todo lo que se compone de electrones y protones con una carga fija explica la cuantización. Pero no explica por qué sólo hay electrones y protones, o por qué la carga del protón es un múltiplo de la del electrón. Lo que la cuantización de Dirac explica a priori , es decir, sin ningún aporte experimental adicional sobre el número de partículas fundamentales cargadas, es que todas las partículas cargadas deben tener una carga que sea un múltiplo de mi , independientemente de que sean fundamentales o compuestos. Esto es diferente de la observación bastante trivial de que las partículas compuestas hechas de dos partículas fundamentales cargadas con cargas mi + = mi se cobran como múltiplos de mi + .

  3. Que otras partículas fundamentales con otros múltiplos enteros de mi pueden existir no significa que deben existir. La cuantización de Dirac solo dice que si existen otras partículas cargadas, deben cuantificarse en términos de la unidad fundamental mi . Tenga en cuenta que, dado que ahora sabemos de la existencia de los quarks, la unidad de carga fundamental de nuestro universo sería un tercio de la carga del electrón. Tenga en cuenta también que el electrón no es compuesto, por lo que su explicación de las cargas de los átomos porque son compuestos de electrón y protón no explica por qué la carga del electrón es un múltiplo entero de la carga del quark (como lo hace con el electrón y el protón). la carga del protón, pero tal vez dos cargas fundamentales iguales podrían verse como naturales, mientras que una relación 1:3 ciertamente exige una explicación de por qué no podría ser irracional).

Aparte, la cuantización de Dirac no es un argumento útil si ya sabe que el grupo de calibre del electromagnetismo es tu ( 1 ) - las representaciones de tu ( 1 ) se clasifican por números enteros, y sus cargas son múltiplos enteros de la carga de la representación fundamental. Pero, clásicamente, es imposible decidir si el grupo de calibre del electromagnetismo es R o tu ( 1 ) . El argumento de Dirac en lenguaje moderno esencialmente muestra que si existe el efecto Aharonov-Bohm y si existen monopolos magnéticos, entonces el grupo de calibre del electromagnetismo cuántico debe ser tu ( 1 ) , no R , si ha de ser coherente.